The Art and Craft of Problem Solving

(Ann) #1

354 CHAPTER 9 CALCULUS


9.4. 19 (Putnam 1997) Evaluate

r[(x_X^3 +�_�+ ... )
Jo^2 2·4^2 ·4·^6

x (^1 + �� + 2 t 42 + 22. �. 62 + .. -

) ] dx.

9.4.20 (Putnam 1990) Is there an infinite sequence
ao, a I , a2,. .. of nonzero real numbers such that for
n = 1,2,3, ... the polynomial
Pn(x) = ao +alx+a 2 ..? + ... +an�
has exactly n distinct real roots?
9.4.2 1 Prove that
Si;X =
Dcos(;),

(a) using telescoping;
(b) using power series.
9.4.22 In Example 9.4.9, U(n,k) was defined by a
nasty integral. Directly evaluate this integral and show
that it equals 1/ (n + 1) using
(a) repeated applications of integration by parts;
(b) manipulation of the binomial series.

Eulerian Mathematics and Number Theory
The following challenging problems are somewhat in­
terrelated, all involving manipulations similar to Ex­
ample 9.4.7. You may need to reread the combina­
torics and number theory chapters, and some familiar­
ity with probability is helpful for the last two prob­
lems. (For the definition of " q" Jl, cr, see pages 162 ,
236, 238, 235, respectively.)
9.4.23 Evaluate
1 1 1 I
12 + 32 + 53 + 72 + ....
9.4.24 Let S be the set of integers whose prime factor­
izations include only the primes 3, 5, and 7. Does the
sum of the reciprocals of the elements of S converge,
and if so, to what?
9.4.25 Consider the argument used in Example 9.4.7.
(a) Did this argument really require the funda­
mental theorem of arithmetic (unique factoriza­
tion)?
(b) Make this argument rigorous, by considering
only finite partial sums and products.

pS
9.4.26 Show that '(s) = n ..-=--s.
p pnme P
9.4.27 Compute
('(2) -1)+('(3)-1)+('(4) -1)+···.
9.4.28 Let P = {4,8,9, 16,... } be the set of perfect
powers, i.e., the set of positive integers of the form ab,
where a and b are integers greater than I. Prove that
1
J�j-l

=l
.

9.4.29 Modify the argument used in Example 9.4.8 to
show that ,(4) = n4/90. Can you generalize this to
find a formula for '(2n)? How about ,(2n + I)?
9.4.30 Find a sequence n I, n 2 , ... such that

9.4.3 1 Prove that

9.4.32 Use the ideas from Problem 9.2.37 and Exam­
ple 9.4.7 to show that, for any positive integer n, the
sum of the reciprocals of the prime numbers that do
not exceed n is greater than log(logn)) - 1/2. Use this
to show that 4 ..!.. diverges.
p pnme P
9.4.33 Fix a positive integer n. Let PI, ... , Pk be
the primes less than or equal to..;n. Let Qn := PI.
P 2 ··· Pk· Let n(x) denote the number of primes less
than or equal to x. Show that

n(n) = -I + n(..;n) + � Jl(d) l� J.
diQn
9.4.34 For n E N,x E R, define q,(n,x) to equal the
number of positive integers less than or equal to x that
are relatively prime to n. For example, q,(n,n) is just
plain old q,(n). Find a formula for computing q,(n,x).
9.4.35 Show that the number of pairs (x,y), where x
and y are relatively prime integers between 1 and n in­
clusive, is

9.4.36 Show that the probability that two randomly
chosen positive integers are relatively prime is
6
2.
n
Free download pdf